Limit of (3^(sin(t))-1)/t as t approaches 0: L'hopital's problem

  • Thread starter SkyChurch
  • Start date
  • Tags
    l'hopital
In summary, the conversation is about finding the limit as t approaches 0 of [(3^(sin(t)))-1]/t and using l'Hopital's rule to do so. The conversation also includes some discussion about the derivative of 3^(sin(t)) and the relationship between e and ln. The conversation ends with a suggestion to review Algebra before continuing with Calculus.
  • #1
SkyChurch

Homework Statement



I need to find the lim as t approaches 0 of [(3^(sin(t)))-1]/t

Homework Equations





The Attempt at a Solution


I have no idea how to go about solving this. Whenever natural logs and "e" are involved I get confused. I know ln is involved here but I don't know how to go about using it. Thanks a lot!
 
Physics news on Phys.org
  • #2
You already said the magic word, l'Hopital. What the derivative of 3^(sin(t))? Remember 3^x=e^(ln(3)*x) and use the chain rule.
 
  • #3
so it would then be [(e^ln3*sint)-1]/t? I still don't know what to do next with e^(ln3*sint)? These ln and e problems really get me. How would I derive that term?
 
  • #4
You should know what the derivative of [tex] e^{f(x)} [/tex] is if you are working with L'Hopitals.

If you do not:
[tex]g(x) = e^{f(x)} \Rightarrow g'(x) = f'(x) e^{f(x)} [/tex]
 
  • #5
Are you allowed to just give me the first derivative to that one term on these forums? Because that's the best way I learn things. I look at the derivative and can then just kind of figure out how you got there on my own and if I can't then I'll just ask questions, but I'm really not getting this one. Like I said natural logs and e give me a ton of trouble.
 
  • #6
I won't speak for others but I don't think that's the best way to learn because on the exam you won't have that privilege. You can always buy REA Problem Solvers, Schaum's Outline, Calculus for Dummies, etc. Those books are GREAT if you learn by example as they have tons of them. Dick told you that

[tex] 3^{sin(t)} = e^{ln(3^{sin(t)})} = e^{sin(t) \cdot ln(3)} [/tex]

And I told you the rule to differentiate so where are you getting stuck?
 
  • #7
I'm getting stuck at deriving ln(3). I'm starting calc II right now so this stuff isn't very fresh in my mind.
 
  • #8
ln(3) is a constant.
 
  • #9
Thanks a ton NME, I actually have calc for dummies but after a quick look through the workbook I couldn't find anything raised to the power of a trig function. I might be asking several more questions here in the upcoming days. I hate not understanding something, calc I didn't deal with ln and e enough for me to feel comfortable with those problems.
 
  • #10
I would strongly suggest reviewing Algebra before you continue on. Most of Calculus 1 and 2 isn't hard, what trips a lot of people up is/are the hole(s) they have from previous classes. There's another way to do this problem which is equivalent and that's using implicit differentiation, i.e. take ln( ) of both sides.
 
  • #11
Yeah all of calc I was no problem. But I think in college algebra and pre calc they didn't spend enough time explaining the relationship of e and ln. Same with calc I. Every time I see a problem that involves or needs to involve ln or e I cringe.
 

1. What is L'Hopital's rule?

L'Hopital's rule is a mathematical technique used to evaluate limits of indeterminate forms, where both the numerator and denominator approach 0 or infinity. It states that if the limit of the ratio of two functions is indeterminate, then the limit of the ratio of their derivatives is equal to the original limit.

2. Why is L'Hopital's rule useful?

L'Hopital's rule is useful because it allows us to evaluate limits that would otherwise be difficult or impossible to solve. It can also simplify complex limits and make them easier to solve.

3. How do you apply L'Hopital's rule?

To apply L'Hopital's rule, you must first rewrite the limit in the form of an indeterminate form (e.g. 0/0 or infinity/infinity). Then, take the derivatives of both the numerator and denominator and evaluate the limit again. Repeat this process until you get a definite answer or until it becomes clear that the limit does not exist.

4. What is the limit of (3^(sin(t))-1)/t as t approaches 0?

The limit of (3^(sin(t))-1)/t as t approaches 0 is equal to 0. This can be found by applying L'Hopital's rule multiple times until the limit becomes a definite value.

5. Are there any limitations to using L'Hopital's rule?

Yes, there are limitations to using L'Hopital's rule. It can only be applied to indeterminate forms and may not work if the original limit does not exist. Additionally, it may not be the most efficient method for evaluating limits and should only be used as a last resort.

Similar threads

Replies
1
Views
635
  • Calculus and Beyond Homework Help
Replies
1
Views
465
  • Calculus and Beyond Homework Help
Replies
23
Views
1K
  • Calculus and Beyond Homework Help
Replies
8
Views
628
  • Calculus and Beyond Homework Help
Replies
3
Views
2K
  • Calculus and Beyond Homework Help
Replies
21
Views
1K
Replies
2
Views
824
  • Calculus and Beyond Homework Help
Replies
2
Views
872
  • Calculus and Beyond Homework Help
Replies
2
Views
535
  • Calculus and Beyond Homework Help
Replies
3
Views
962
Back
Top